Bạn chưa đăng nhập. Vui lòng đăng nhập để hỏi bài

Những câu hỏi liên quan
Nam Ngô Đăng Hải
Xem chi tiết
Xem chi tiết
Diệp Nguyễn Thị Huyền
Xem chi tiết
Trung Nguyen
Xem chi tiết
trần thành đạt
7 tháng 1 2018 lúc 22:21

A=x^3 +y^3 +z^3+ 2(x/y+z  +y/z+x  +z/x+y)  \(\ge x^3+y^3+z^3+2.\frac{3}{2}\)  (bạn vào tìm BĐT nesbit là sẽ cm cái đằng sau >= 3/2)

Áp dụng cô si \(x^3+y^3+z^3\ge3xyz=3\)

===> A\(\ge3+3=6\) khi x=y=z=1

•๖ۣۜUηĭɗεηтĭƒĭεɗ
Xem chi tiết
Yuzu
16 tháng 8 2019 lúc 21:08

Ta có

\(x+y+z=1\Leftrightarrow\left(x+y+z\right)^2=1\Leftrightarrow\left[\left(x+y\right)+z\right]^2=1\\ \Leftrightarrow1=\left[\left(x+y\right)+z\right]^2\ge4\left(x+y\right)z\left(bđtAM-GM\right)\\ \Leftrightarrow\frac{x+y}{xyz}\ge\frac{4\left(x+y\right)^2z}{xyz}\ge\frac{4\cdot4xy\cdot z}{xyz}=16\)

(nhân cả hai vế với \(\frac{x+y}{xyz}\))

Vậy min A = 16 khi

\(\left\{{}\begin{matrix}x+y=z\\x=y\\x+y+z=1\end{matrix}\right.\Leftrightarrow x=y=\frac{1}{4},z=\frac{1}{2}\)

P.s: Cái chỗ bđt AM-GM bạn có thể thay bằng việc c/m bđt dưới để áp dụng vào bài toán:

\(\left(a+b\right)^2\ge4ab\)

Quách Thị Diệp Chi
Xem chi tiết
Tạ Uyên
Xem chi tiết
Xyz OLM
29 tháng 1 2022 lúc 10:46

Có \(P=\dfrac{x+z}{xyz}=\dfrac{1}{yz}+\dfrac{1}{xy}=\dfrac{1}{y}\left(\dfrac{1}{x}+\dfrac{1}{z}\right)\ge\dfrac{1}{y}.\dfrac{4}{x+z}\)

\(=\dfrac{4}{y\left(x+z\right)}=\dfrac{4}{y\left(4-y\right)}=\dfrac{4}{-y^2+4y}=\dfrac{4}{-\left(y-2\right)^2+4}\ge1\)

"=" xảy ra khi y = 2 ; x = 1 ; z = 1

Tạ Uyên
29 tháng 1 2022 lúc 10:37

Giúp mình câu này với ah.

 

Trần Đức Huy
29 tháng 1 2022 lúc 11:00

Ta có x+y+z=4

=>y=4-x-z

Ta có :x,y,z>0

=>\(x^2>0,z^2>0\)

=>\(x^2z>0,z^2x>0\)

Áp dụng bất đẳng thức cô si với hai số dương \(x^2z\) và z ta có

      \(x^2z+z\)>=2\(\sqrt{x^2z.z}\)

<=>\(x^2z+z>=2xz\)

CMTT:\(z^2x+x>=2xz\)

=>\(x^2z+z+z^2x+x>=4xz\)

=>\(x+z>=4xz-x^2z-z^2x\)

=>\(x+z>=xz\left(4-x-z\right)\)

Mà y=4-x-z(cmt)

=>\(x+z>=xyz\)

=>\(\dfrac{x+z}{xyz}>=1\)

hay \(P>=1\)

Dấu "=" xảy ra <=> \(\left\{{}\begin{matrix}x^2z=z\\z^2x=x\\x+y+z=4\end{matrix}\right.\)

                        <=>\(\left\{{}\begin{matrix}x^2=1\\z^2=1\\x+y+z=4\end{matrix}\right.\)  

                        <=>\(\left\{{}\begin{matrix}x=1\\z=1\\1+y+1=4\end{matrix}\right.\)

                        <=>\(\left\{{}\begin{matrix}x=1\\z=1\\y=2\end{matrix}\right.\)

Vậy tại x=1, y=2,z=1 thì P có giá trị nhỏ nhất là 1

nguyen thi kim dung
Xem chi tiết
Trần Hữu Ngọc Minh
Xem chi tiết